Values of $a$ for which $p(x)$ has a complex roots?












2














Let $p(x) = x^4 + ax^3 + bx^2 + ax + 1$. Suppose $x = 1$ is a root of $p(x)$, then find the range of values of $a$ such that $p(x)$ has complex (non-real) roots?



My approach: Using the fact that $1$ is a root, I am able to deduce $b = -2(a+1)$. Then I wrote $$p(x) = (x-1)(x-z)(x-bar{z})(x-k)$$ where $z = c + id, dneq 0$ and $k$ is some other real roots. Now I plan to expand and compare the coefficients but have been unable to do so in a fruitful manner (there just too many variables to get to a proper estimate of range of $a$ )










share|cite|improve this question



























    2














    Let $p(x) = x^4 + ax^3 + bx^2 + ax + 1$. Suppose $x = 1$ is a root of $p(x)$, then find the range of values of $a$ such that $p(x)$ has complex (non-real) roots?



    My approach: Using the fact that $1$ is a root, I am able to deduce $b = -2(a+1)$. Then I wrote $$p(x) = (x-1)(x-z)(x-bar{z})(x-k)$$ where $z = c + id, dneq 0$ and $k$ is some other real roots. Now I plan to expand and compare the coefficients but have been unable to do so in a fruitful manner (there just too many variables to get to a proper estimate of range of $a$ )










    share|cite|improve this question

























      2












      2








      2


      1





      Let $p(x) = x^4 + ax^3 + bx^2 + ax + 1$. Suppose $x = 1$ is a root of $p(x)$, then find the range of values of $a$ such that $p(x)$ has complex (non-real) roots?



      My approach: Using the fact that $1$ is a root, I am able to deduce $b = -2(a+1)$. Then I wrote $$p(x) = (x-1)(x-z)(x-bar{z})(x-k)$$ where $z = c + id, dneq 0$ and $k$ is some other real roots. Now I plan to expand and compare the coefficients but have been unable to do so in a fruitful manner (there just too many variables to get to a proper estimate of range of $a$ )










      share|cite|improve this question













      Let $p(x) = x^4 + ax^3 + bx^2 + ax + 1$. Suppose $x = 1$ is a root of $p(x)$, then find the range of values of $a$ such that $p(x)$ has complex (non-real) roots?



      My approach: Using the fact that $1$ is a root, I am able to deduce $b = -2(a+1)$. Then I wrote $$p(x) = (x-1)(x-z)(x-bar{z})(x-k)$$ where $z = c + id, dneq 0$ and $k$ is some other real roots. Now I plan to expand and compare the coefficients but have been unable to do so in a fruitful manner (there just too many variables to get to a proper estimate of range of $a$ )







      polynomials






      share|cite|improve this question













      share|cite|improve this question











      share|cite|improve this question




      share|cite|improve this question










      asked Nov 28 '18 at 1:37









      henceproved

      1358




      1358






















          4 Answers
          4






          active

          oldest

          votes


















          4














          HINT:



          $$p(x)=(x-1)(x^3+(a+1)x^2-(a+1)x-1)$$



          Rest follows very nicely.






          share|cite|improve this answer





















          • If you don't see how, theres a solution to $$x^3+(a+1)x^2-(a+1)x-1=0$$ that you should be able to spot immediately.
            – Rhys Hughes
            Nov 28 '18 at 2:03










          • $1$ is a mutiple root and after that I have to show that determinant is $leq 0$ for the remaining quadritic term.
            – henceproved
            Nov 28 '18 at 2:09










          • Absolutely, except the inequality is strict $(<0$ rather than $le 0)$
            – Rhys Hughes
            Nov 28 '18 at 5:09



















          1














          When a polynomial is "palindromic", reading the same way forwards and backwards, it must also have palindromic factors. Thus



          $x^4+ax^3+bx^2+ax+1=(x^2+cx+1)(x^2+dx+1)$



          If $x=1$ is to be a root then it must give $c=-2$ or $d=-2$ in one of the factors, as no other choice has the root $x=1$. Taking $c$ as $-2$ we then get the complex roots if $-2<d<2$, corresponding to a negative discriminate for the $x^2+dx+1$ factor.



          Expanding the product $(x^2+cx+1)(x^2+dx+1)$ and equating this to $x^4+ax^3+bx^2+ax+1$ then implies $a=c+d=d-2$ so the solution to the problem is $-4<a<0$.






          share|cite|improve this answer





























            0














            Here's a different approach. (I can't say for sure whether you can or cannot do it by comparing coefficients though!)



            For a degree 2 polynomial, we know that it has complex roots (assuming the coefficients are real) iff the discriminant is negative. There's a similar generalisation for high degree polynomial.



            In particular, a degree 4 polynomial
            $$ax^4+bx^3+cx^2+dx+e$$
            has discriminant
            $$256a^3e^3-192a^2bde^2-128a^2c^2e^2+144a^2d^2e...$$



            (see https://en.wikipedia.org/wiki/Discriminant#Degree_4 for the full formula). And we know that since $x=1$ is a root there can only be a pair of complex roots so we require the discriminant to be negative.






            share|cite|improve this answer





















            • I doubt this approach is feasible as I will be solving sixth degree $p(a)< 0$.
              – henceproved
              Nov 28 '18 at 1:56












            • Actually your polynomial always has a double root so the discriminant is always 0 for all $a$ (from which you can work from there). But anyway the above answer is definitely better.
              – Rcwt
              Nov 28 '18 at 2:04





















            0














            Considering that $$p(x) = x^4 + ax^3 + bx^2 + ax + 1$$ If $x=1$ is a root, then
            $$p(1)=2 a+b+2=0 implies b=-2-2a$$ and
            $$p(x)=(x-1),(x^3+(a+1) x^2-(a+1) x-1)$$ Clearly, $x=1$ is also a root for the second term making
            $$p(x)=(x-1)^2,(x^2+(a+2) x+1)$$ So, there are complex roots if the discriminant $Delta$ of the remaining quadratic equation is negative.






            share|cite|improve this answer





















              Your Answer





              StackExchange.ifUsing("editor", function () {
              return StackExchange.using("mathjaxEditing", function () {
              StackExchange.MarkdownEditor.creationCallbacks.add(function (editor, postfix) {
              StackExchange.mathjaxEditing.prepareWmdForMathJax(editor, postfix, [["$", "$"], ["\\(","\\)"]]);
              });
              });
              }, "mathjax-editing");

              StackExchange.ready(function() {
              var channelOptions = {
              tags: "".split(" "),
              id: "69"
              };
              initTagRenderer("".split(" "), "".split(" "), channelOptions);

              StackExchange.using("externalEditor", function() {
              // Have to fire editor after snippets, if snippets enabled
              if (StackExchange.settings.snippets.snippetsEnabled) {
              StackExchange.using("snippets", function() {
              createEditor();
              });
              }
              else {
              createEditor();
              }
              });

              function createEditor() {
              StackExchange.prepareEditor({
              heartbeatType: 'answer',
              autoActivateHeartbeat: false,
              convertImagesToLinks: true,
              noModals: true,
              showLowRepImageUploadWarning: true,
              reputationToPostImages: 10,
              bindNavPrevention: true,
              postfix: "",
              imageUploader: {
              brandingHtml: "Powered by u003ca class="icon-imgur-white" href="https://imgur.com/"u003eu003c/au003e",
              contentPolicyHtml: "User contributions licensed under u003ca href="https://creativecommons.org/licenses/by-sa/3.0/"u003ecc by-sa 3.0 with attribution requiredu003c/au003e u003ca href="https://stackoverflow.com/legal/content-policy"u003e(content policy)u003c/au003e",
              allowUrls: true
              },
              noCode: true, onDemand: true,
              discardSelector: ".discard-answer"
              ,immediatelyShowMarkdownHelp:true
              });


              }
              });














              draft saved

              draft discarded


















              StackExchange.ready(
              function () {
              StackExchange.openid.initPostLogin('.new-post-login', 'https%3a%2f%2fmath.stackexchange.com%2fquestions%2f3016588%2fvalues-of-a-for-which-px-has-a-complex-roots%23new-answer', 'question_page');
              }
              );

              Post as a guest















              Required, but never shown

























              4 Answers
              4






              active

              oldest

              votes








              4 Answers
              4






              active

              oldest

              votes









              active

              oldest

              votes






              active

              oldest

              votes









              4














              HINT:



              $$p(x)=(x-1)(x^3+(a+1)x^2-(a+1)x-1)$$



              Rest follows very nicely.






              share|cite|improve this answer





















              • If you don't see how, theres a solution to $$x^3+(a+1)x^2-(a+1)x-1=0$$ that you should be able to spot immediately.
                – Rhys Hughes
                Nov 28 '18 at 2:03










              • $1$ is a mutiple root and after that I have to show that determinant is $leq 0$ for the remaining quadritic term.
                – henceproved
                Nov 28 '18 at 2:09










              • Absolutely, except the inequality is strict $(<0$ rather than $le 0)$
                – Rhys Hughes
                Nov 28 '18 at 5:09
















              4














              HINT:



              $$p(x)=(x-1)(x^3+(a+1)x^2-(a+1)x-1)$$



              Rest follows very nicely.






              share|cite|improve this answer





















              • If you don't see how, theres a solution to $$x^3+(a+1)x^2-(a+1)x-1=0$$ that you should be able to spot immediately.
                – Rhys Hughes
                Nov 28 '18 at 2:03










              • $1$ is a mutiple root and after that I have to show that determinant is $leq 0$ for the remaining quadritic term.
                – henceproved
                Nov 28 '18 at 2:09










              • Absolutely, except the inequality is strict $(<0$ rather than $le 0)$
                – Rhys Hughes
                Nov 28 '18 at 5:09














              4












              4








              4






              HINT:



              $$p(x)=(x-1)(x^3+(a+1)x^2-(a+1)x-1)$$



              Rest follows very nicely.






              share|cite|improve this answer












              HINT:



              $$p(x)=(x-1)(x^3+(a+1)x^2-(a+1)x-1)$$



              Rest follows very nicely.







              share|cite|improve this answer












              share|cite|improve this answer



              share|cite|improve this answer










              answered Nov 28 '18 at 1:58









              Rhys Hughes

              4,8041327




              4,8041327












              • If you don't see how, theres a solution to $$x^3+(a+1)x^2-(a+1)x-1=0$$ that you should be able to spot immediately.
                – Rhys Hughes
                Nov 28 '18 at 2:03










              • $1$ is a mutiple root and after that I have to show that determinant is $leq 0$ for the remaining quadritic term.
                – henceproved
                Nov 28 '18 at 2:09










              • Absolutely, except the inequality is strict $(<0$ rather than $le 0)$
                – Rhys Hughes
                Nov 28 '18 at 5:09


















              • If you don't see how, theres a solution to $$x^3+(a+1)x^2-(a+1)x-1=0$$ that you should be able to spot immediately.
                – Rhys Hughes
                Nov 28 '18 at 2:03










              • $1$ is a mutiple root and after that I have to show that determinant is $leq 0$ for the remaining quadritic term.
                – henceproved
                Nov 28 '18 at 2:09










              • Absolutely, except the inequality is strict $(<0$ rather than $le 0)$
                – Rhys Hughes
                Nov 28 '18 at 5:09
















              If you don't see how, theres a solution to $$x^3+(a+1)x^2-(a+1)x-1=0$$ that you should be able to spot immediately.
              – Rhys Hughes
              Nov 28 '18 at 2:03




              If you don't see how, theres a solution to $$x^3+(a+1)x^2-(a+1)x-1=0$$ that you should be able to spot immediately.
              – Rhys Hughes
              Nov 28 '18 at 2:03












              $1$ is a mutiple root and after that I have to show that determinant is $leq 0$ for the remaining quadritic term.
              – henceproved
              Nov 28 '18 at 2:09




              $1$ is a mutiple root and after that I have to show that determinant is $leq 0$ for the remaining quadritic term.
              – henceproved
              Nov 28 '18 at 2:09












              Absolutely, except the inequality is strict $(<0$ rather than $le 0)$
              – Rhys Hughes
              Nov 28 '18 at 5:09




              Absolutely, except the inequality is strict $(<0$ rather than $le 0)$
              – Rhys Hughes
              Nov 28 '18 at 5:09











              1














              When a polynomial is "palindromic", reading the same way forwards and backwards, it must also have palindromic factors. Thus



              $x^4+ax^3+bx^2+ax+1=(x^2+cx+1)(x^2+dx+1)$



              If $x=1$ is to be a root then it must give $c=-2$ or $d=-2$ in one of the factors, as no other choice has the root $x=1$. Taking $c$ as $-2$ we then get the complex roots if $-2<d<2$, corresponding to a negative discriminate for the $x^2+dx+1$ factor.



              Expanding the product $(x^2+cx+1)(x^2+dx+1)$ and equating this to $x^4+ax^3+bx^2+ax+1$ then implies $a=c+d=d-2$ so the solution to the problem is $-4<a<0$.






              share|cite|improve this answer


























                1














                When a polynomial is "palindromic", reading the same way forwards and backwards, it must also have palindromic factors. Thus



                $x^4+ax^3+bx^2+ax+1=(x^2+cx+1)(x^2+dx+1)$



                If $x=1$ is to be a root then it must give $c=-2$ or $d=-2$ in one of the factors, as no other choice has the root $x=1$. Taking $c$ as $-2$ we then get the complex roots if $-2<d<2$, corresponding to a negative discriminate for the $x^2+dx+1$ factor.



                Expanding the product $(x^2+cx+1)(x^2+dx+1)$ and equating this to $x^4+ax^3+bx^2+ax+1$ then implies $a=c+d=d-2$ so the solution to the problem is $-4<a<0$.






                share|cite|improve this answer
























                  1












                  1








                  1






                  When a polynomial is "palindromic", reading the same way forwards and backwards, it must also have palindromic factors. Thus



                  $x^4+ax^3+bx^2+ax+1=(x^2+cx+1)(x^2+dx+1)$



                  If $x=1$ is to be a root then it must give $c=-2$ or $d=-2$ in one of the factors, as no other choice has the root $x=1$. Taking $c$ as $-2$ we then get the complex roots if $-2<d<2$, corresponding to a negative discriminate for the $x^2+dx+1$ factor.



                  Expanding the product $(x^2+cx+1)(x^2+dx+1)$ and equating this to $x^4+ax^3+bx^2+ax+1$ then implies $a=c+d=d-2$ so the solution to the problem is $-4<a<0$.






                  share|cite|improve this answer












                  When a polynomial is "palindromic", reading the same way forwards and backwards, it must also have palindromic factors. Thus



                  $x^4+ax^3+bx^2+ax+1=(x^2+cx+1)(x^2+dx+1)$



                  If $x=1$ is to be a root then it must give $c=-2$ or $d=-2$ in one of the factors, as no other choice has the root $x=1$. Taking $c$ as $-2$ we then get the complex roots if $-2<d<2$, corresponding to a negative discriminate for the $x^2+dx+1$ factor.



                  Expanding the product $(x^2+cx+1)(x^2+dx+1)$ and equating this to $x^4+ax^3+bx^2+ax+1$ then implies $a=c+d=d-2$ so the solution to the problem is $-4<a<0$.







                  share|cite|improve this answer












                  share|cite|improve this answer



                  share|cite|improve this answer










                  answered Nov 28 '18 at 2:04









                  Oscar Lanzi

                  12.1k12036




                  12.1k12036























                      0














                      Here's a different approach. (I can't say for sure whether you can or cannot do it by comparing coefficients though!)



                      For a degree 2 polynomial, we know that it has complex roots (assuming the coefficients are real) iff the discriminant is negative. There's a similar generalisation for high degree polynomial.



                      In particular, a degree 4 polynomial
                      $$ax^4+bx^3+cx^2+dx+e$$
                      has discriminant
                      $$256a^3e^3-192a^2bde^2-128a^2c^2e^2+144a^2d^2e...$$



                      (see https://en.wikipedia.org/wiki/Discriminant#Degree_4 for the full formula). And we know that since $x=1$ is a root there can only be a pair of complex roots so we require the discriminant to be negative.






                      share|cite|improve this answer





















                      • I doubt this approach is feasible as I will be solving sixth degree $p(a)< 0$.
                        – henceproved
                        Nov 28 '18 at 1:56












                      • Actually your polynomial always has a double root so the discriminant is always 0 for all $a$ (from which you can work from there). But anyway the above answer is definitely better.
                        – Rcwt
                        Nov 28 '18 at 2:04


















                      0














                      Here's a different approach. (I can't say for sure whether you can or cannot do it by comparing coefficients though!)



                      For a degree 2 polynomial, we know that it has complex roots (assuming the coefficients are real) iff the discriminant is negative. There's a similar generalisation for high degree polynomial.



                      In particular, a degree 4 polynomial
                      $$ax^4+bx^3+cx^2+dx+e$$
                      has discriminant
                      $$256a^3e^3-192a^2bde^2-128a^2c^2e^2+144a^2d^2e...$$



                      (see https://en.wikipedia.org/wiki/Discriminant#Degree_4 for the full formula). And we know that since $x=1$ is a root there can only be a pair of complex roots so we require the discriminant to be negative.






                      share|cite|improve this answer





















                      • I doubt this approach is feasible as I will be solving sixth degree $p(a)< 0$.
                        – henceproved
                        Nov 28 '18 at 1:56












                      • Actually your polynomial always has a double root so the discriminant is always 0 for all $a$ (from which you can work from there). But anyway the above answer is definitely better.
                        – Rcwt
                        Nov 28 '18 at 2:04
















                      0












                      0








                      0






                      Here's a different approach. (I can't say for sure whether you can or cannot do it by comparing coefficients though!)



                      For a degree 2 polynomial, we know that it has complex roots (assuming the coefficients are real) iff the discriminant is negative. There's a similar generalisation for high degree polynomial.



                      In particular, a degree 4 polynomial
                      $$ax^4+bx^3+cx^2+dx+e$$
                      has discriminant
                      $$256a^3e^3-192a^2bde^2-128a^2c^2e^2+144a^2d^2e...$$



                      (see https://en.wikipedia.org/wiki/Discriminant#Degree_4 for the full formula). And we know that since $x=1$ is a root there can only be a pair of complex roots so we require the discriminant to be negative.






                      share|cite|improve this answer












                      Here's a different approach. (I can't say for sure whether you can or cannot do it by comparing coefficients though!)



                      For a degree 2 polynomial, we know that it has complex roots (assuming the coefficients are real) iff the discriminant is negative. There's a similar generalisation for high degree polynomial.



                      In particular, a degree 4 polynomial
                      $$ax^4+bx^3+cx^2+dx+e$$
                      has discriminant
                      $$256a^3e^3-192a^2bde^2-128a^2c^2e^2+144a^2d^2e...$$



                      (see https://en.wikipedia.org/wiki/Discriminant#Degree_4 for the full formula). And we know that since $x=1$ is a root there can only be a pair of complex roots so we require the discriminant to be negative.







                      share|cite|improve this answer












                      share|cite|improve this answer



                      share|cite|improve this answer










                      answered Nov 28 '18 at 1:50









                      Rcwt

                      3871210




                      3871210












                      • I doubt this approach is feasible as I will be solving sixth degree $p(a)< 0$.
                        – henceproved
                        Nov 28 '18 at 1:56












                      • Actually your polynomial always has a double root so the discriminant is always 0 for all $a$ (from which you can work from there). But anyway the above answer is definitely better.
                        – Rcwt
                        Nov 28 '18 at 2:04




















                      • I doubt this approach is feasible as I will be solving sixth degree $p(a)< 0$.
                        – henceproved
                        Nov 28 '18 at 1:56












                      • Actually your polynomial always has a double root so the discriminant is always 0 for all $a$ (from which you can work from there). But anyway the above answer is definitely better.
                        – Rcwt
                        Nov 28 '18 at 2:04


















                      I doubt this approach is feasible as I will be solving sixth degree $p(a)< 0$.
                      – henceproved
                      Nov 28 '18 at 1:56






                      I doubt this approach is feasible as I will be solving sixth degree $p(a)< 0$.
                      – henceproved
                      Nov 28 '18 at 1:56














                      Actually your polynomial always has a double root so the discriminant is always 0 for all $a$ (from which you can work from there). But anyway the above answer is definitely better.
                      – Rcwt
                      Nov 28 '18 at 2:04






                      Actually your polynomial always has a double root so the discriminant is always 0 for all $a$ (from which you can work from there). But anyway the above answer is definitely better.
                      – Rcwt
                      Nov 28 '18 at 2:04













                      0














                      Considering that $$p(x) = x^4 + ax^3 + bx^2 + ax + 1$$ If $x=1$ is a root, then
                      $$p(1)=2 a+b+2=0 implies b=-2-2a$$ and
                      $$p(x)=(x-1),(x^3+(a+1) x^2-(a+1) x-1)$$ Clearly, $x=1$ is also a root for the second term making
                      $$p(x)=(x-1)^2,(x^2+(a+2) x+1)$$ So, there are complex roots if the discriminant $Delta$ of the remaining quadratic equation is negative.






                      share|cite|improve this answer


























                        0














                        Considering that $$p(x) = x^4 + ax^3 + bx^2 + ax + 1$$ If $x=1$ is a root, then
                        $$p(1)=2 a+b+2=0 implies b=-2-2a$$ and
                        $$p(x)=(x-1),(x^3+(a+1) x^2-(a+1) x-1)$$ Clearly, $x=1$ is also a root for the second term making
                        $$p(x)=(x-1)^2,(x^2+(a+2) x+1)$$ So, there are complex roots if the discriminant $Delta$ of the remaining quadratic equation is negative.






                        share|cite|improve this answer
























                          0












                          0








                          0






                          Considering that $$p(x) = x^4 + ax^3 + bx^2 + ax + 1$$ If $x=1$ is a root, then
                          $$p(1)=2 a+b+2=0 implies b=-2-2a$$ and
                          $$p(x)=(x-1),(x^3+(a+1) x^2-(a+1) x-1)$$ Clearly, $x=1$ is also a root for the second term making
                          $$p(x)=(x-1)^2,(x^2+(a+2) x+1)$$ So, there are complex roots if the discriminant $Delta$ of the remaining quadratic equation is negative.






                          share|cite|improve this answer












                          Considering that $$p(x) = x^4 + ax^3 + bx^2 + ax + 1$$ If $x=1$ is a root, then
                          $$p(1)=2 a+b+2=0 implies b=-2-2a$$ and
                          $$p(x)=(x-1),(x^3+(a+1) x^2-(a+1) x-1)$$ Clearly, $x=1$ is also a root for the second term making
                          $$p(x)=(x-1)^2,(x^2+(a+2) x+1)$$ So, there are complex roots if the discriminant $Delta$ of the remaining quadratic equation is negative.







                          share|cite|improve this answer












                          share|cite|improve this answer



                          share|cite|improve this answer










                          answered Nov 28 '18 at 3:53









                          Claude Leibovici

                          119k1157132




                          119k1157132






























                              draft saved

                              draft discarded




















































                              Thanks for contributing an answer to Mathematics Stack Exchange!


                              • Please be sure to answer the question. Provide details and share your research!

                              But avoid



                              • Asking for help, clarification, or responding to other answers.

                              • Making statements based on opinion; back them up with references or personal experience.


                              Use MathJax to format equations. MathJax reference.


                              To learn more, see our tips on writing great answers.





                              Some of your past answers have not been well-received, and you're in danger of being blocked from answering.


                              Please pay close attention to the following guidance:


                              • Please be sure to answer the question. Provide details and share your research!

                              But avoid



                              • Asking for help, clarification, or responding to other answers.

                              • Making statements based on opinion; back them up with references or personal experience.


                              To learn more, see our tips on writing great answers.




                              draft saved


                              draft discarded














                              StackExchange.ready(
                              function () {
                              StackExchange.openid.initPostLogin('.new-post-login', 'https%3a%2f%2fmath.stackexchange.com%2fquestions%2f3016588%2fvalues-of-a-for-which-px-has-a-complex-roots%23new-answer', 'question_page');
                              }
                              );

                              Post as a guest















                              Required, but never shown





















































                              Required, but never shown














                              Required, but never shown












                              Required, but never shown







                              Required, but never shown

































                              Required, but never shown














                              Required, but never shown












                              Required, but never shown







                              Required, but never shown







                              Popular posts from this blog

                              How do I know what Microsoft account the skydrive app is syncing to?

                              When does type information flow backwards in C++?

                              Grease: Live!